Rational numbers and Lowest terms proof:

In summary, the conversation discusses two problems related to fractions and their lowest terms. The first problem involves proving that if two fractions are equal in lowest terms, then their numerators and denominators must also be equal. The second problem involves proving that a fraction can only be an integer if its denominator is equal to 1. The conversation also includes tips and hints for solving the problems, such as using prime factorization and the concept of contradiction.
  • #1
silvermane
Gold Member
117
0
I've been recently reading a book on abstract algebra and number theory, and I stumbled upon a problem that at first glance looked obvious, but I can't seem to figure out how to formally write the proof.

1.)So, let's say we have 4 integers, r,s,t,u, all greater than or equal to 1. Suppose [tex]\frac{r}{s}[/tex] = [tex]\frac{t}{u}[/tex] where both fractions are in lowest terms. Prove that r=t and s=u.

For this problem, I was thinking of solving it via contradiction but I can't seem to get there. I was thinking of using the fact that the gcd(r,s) = 1, and if the gcd(t,u)=1 that there would be a contradiction if they were not equal, but I feel like that's not enough. Any tips or hints would be greatly appreciated.

2.) Now, suppose we have r and s again (lowest terms), and we look at [tex]\frac{r}{s}[/tex]. Prove that an integer N cannot equal [tex]\frac{r}{s}[/tex] unless s = 1.

I was thinking of saying that we can write r as a product of primes:
r=[tex]p_{1}[/tex]*[tex]p_{2}[/tex]*...*[tex]p_{k}[/tex]​

and then writing s as a product of primes, but primes that are all different from r's:
s=[tex]q_{1}[/tex]*[tex]q_{2}[/tex]*...*[tex]q_{k}[/tex]​

Obviously through some algebraic manipulation, we see that [tex]\frac{r}{s}[/tex] is also in lowest terms, and thus can't be an integer unless s=1, but I feel like I need more detail in this part of my proof.

Once again, thank you all in advance for your help and advice :)
 
Physics news on Phys.org
  • #2
You're on the right track.

Let [tex] r = p_1 * p_2 * ... * p_a. [/tex]

Let [tex] s = q_1 * q_2 * ... * q_b. [/tex]

Let [tex] t = p'_1 * p'_2 * ... * p'_c. [/tex]

Let [tex] u = q'_1 * q'_2 * ... * q'_d. [/tex]

Cross multiply to get

[tex] (p_1 * p_2 * ... * p_k)(q'_1 * q'_2 * ... * q'_k) = (p'_1 * p'_2 * ... * p'_k)(q_1 * q_2 * ... * q_k) .[/tex]

We know that [tex] p_i \neq q_k [/tex] and [tex] p'_i \neq q'_k \ [/tex] for all i and k.

Can you figure out the rest?
 
  • #3
Raskolnikov said:
You're on the right track.

Let [tex] r = p_1 * p_2 * ... * p_a. [/tex]

Let [tex] s = q_1 * q_2 * ... * q_b. [/tex]

Let [tex] t = p'_1 * p'_2 * ... * p'_c. [/tex]

Let [tex] u = q'_1 * q'_2 * ... * q'_d. [/tex]

Cross multiply to get

[tex] (p_1 * p_2 * ... * p_k)(q'_1 * q'_2 * ... * q'_k) = (p'_1 * p'_2 * ... * p'_k)(q_1 * q_2 * ... * q_k) .[/tex]

We know that [tex] p_i \neq q_k [/tex] and [tex] p'_i \neq q'_k \ [/tex] for all i and k.

Can you figure out the rest?


Yes, very much so! You've been more than helpful. I thought of doing it that way, but must have been distracted somehow. Thank you for all your help! :)
 
  • #4
For the second one is it not enough to prove by contradiction?

Like suppose [tex]\frac{r}{s}=N[/tex] then s divides r which leads to a contradiction ,unless s=1, given our choice of s and r namely gcd(r,s)=1.

Is this what you are doing ?
 
  • #5
╔(σ_σ)╝ said:
For the second one is it not enough to prove by contradiction?

Like suppose [tex]\frac{r}{s}=N[/tex] then s divides r which leads to a contradiction ,unless s=1, given our choice of s and r namely gcd(r,s)=1.

Is this what you are doing ?

Yes that's exactly what I've done. I just needed to know that I had to break them all into a product of irreducible primes to better understand what was going on. Thank you for your help :)
 

Related to Rational numbers and Lowest terms proof:

What are rational numbers?

Rational numbers are numbers that can be expressed as a ratio of two integers, where the denominator is not zero. They can be written in the form a/b, where a and b are integers and b is not equal to zero. Examples of rational numbers include 1/2, 5/3, and -2/7.

What is the proof for lowest terms?

The proof for lowest terms states that every rational number has a unique representation in lowest terms, meaning that the numerator and denominator of the fraction have no common factors other than 1. This can be proven using the fundamental theorem of arithmetic, which states that every integer greater than 1 can be written as a product of prime numbers in a unique way.

How do you simplify rational numbers?

To simplify a rational number, you need to find the greatest common factor (GCF) of the numerator and denominator and divide both numbers by the GCF. This will result in a fraction in lowest terms. For example, to simplify 6/12, the GCF of 6 and 12 is 6, so the simplified fraction is 1/2.

Can irrational numbers be expressed in lowest terms?

No, irrational numbers cannot be expressed in lowest terms because they cannot be written as a ratio of two integers. Irrational numbers, such as pi or the square root of 2, have non-repeating and non-terminating decimal representations and cannot be simplified to a fraction with integer values.

Why is it important to prove lowest terms?

Proving lowest terms is important because it ensures that every rational number has a unique and simplified representation. This makes it easier to compare and perform operations with rational numbers. Additionally, it helps to identify patterns and relationships between fractions, making them easier to understand and work with.

Similar threads

  • Calculus and Beyond Homework Help
Replies
5
Views
2K
  • Calculus and Beyond Homework Help
Replies
13
Views
2K
  • Calculus and Beyond Homework Help
Replies
4
Views
1K
  • Calculus and Beyond Homework Help
Replies
1
Views
621
  • Calculus and Beyond Homework Help
Replies
1
Views
603
  • Calculus and Beyond Homework Help
Replies
20
Views
2K
  • Calculus and Beyond Homework Help
Replies
12
Views
305
  • Calculus and Beyond Homework Help
Replies
12
Views
1K
  • Calculus and Beyond Homework Help
Replies
5
Views
1K
  • Calculus and Beyond Homework Help
Replies
3
Views
584
Back
Top